Selection of an interval affects the energy eigenstates - impossible

In summary, the discussion is about deriving an equation for the energy levels of a particle in an infinite potential well in two different ways. The first method, using the interval of 0<x<d, results in an equation that matches with existing books. However, the second method, using the interval of -d/2<x<d/2, results in a different equation for energy levels, which is impossible as the energies should not depend on the interval chosen. The derivation and solutions are provided for both methods, with the conclusion that the two boundary conditions used are redundant due to the symmetry of the potential function, and the allowed values of energy are the same for both methods.
  • #1
71GA
208
0
I haven't found a comparison like this in any book that I have been reading so let me explain.

I decided that I will derive an equation for an energy levels of a particle in an infinite potential well in two ways. 1st I tried to derive it using the interval ##0<x<d## where ##d## is a width of a well. What I got was an equation which matches perfectly with any of the books out there:

\begin{align}
&\boxed{E=\frac{N^2\pi^2\hbar^2}{2md^2}}\longrightarrow N=1,2,3\dots\\
&E_1=\pi^2\hbar^2/2md^2\\
&E_2=2^2E_1\\
&E_3=3^2E_1\\
&\dots
\end{align}

So far so good. Now I decided to use the interval ##-d/2<x<d/2## and what I got was a different equation for energy levels. This is impossible because energies can't depend on our selection of an interval. I included my derivation:

\begin{align}
\frac{d^2\psi}{dx^2} &= - \left(\frac{2mE}{\hbar^2}\right)\psi \longleftarrow \substack{\text{Diferential eq. (DE) I get if I}\\\text{set $V=0$ in a Schrodinger. eq. }}\\
&\downarrow\\
\psi &= A\sin\left(\sqrt{\tfrac{2mE}{\hbar^2}}x\right)+B \cos\left(\sqrt{\tfrac{2mE}{\hbar^2}}x\right)\longleftarrow \substack{\text{A general solution}\\\text{to the above DE}}
\end{align}

Let us not forget ##\psi## is in fact ##\psi(x)##. At this point I used the border equations ##\psi(-d/2)=0## and ##\psi(d/2)=0## and what I got was a system of two equations:

\begin{align}
0 &= A\sin\left(\sqrt{\tfrac{2mE}{\hbar^2}} \left(-\tfrac{d}{2}\right)\right)+B \cos\left(\sqrt{\tfrac{2mE}{\hbar^2}}\left(-\tfrac{d}{2}\right)\right)\\
\substack{\text{What I get if I insert $x=-\frac{d}{2}$}}\longrightarrow 0 &= -A\sin\left(\sqrt{\tfrac{2mE}{\hbar^2}} \tfrac{d}{2}\right)+B \cos\left(\sqrt{\tfrac{2mE}{\hbar^2}}\tfrac{d}{2}\right)\\
\phantom{d}\\
\substack{\text{What I get if I insert $x=\frac{d}{2}$}}\longrightarrow 0 &=A\sin\left(\sqrt{\tfrac{2mE}{\hbar^2}} \tfrac{d}{2}\right)+B \cos\left(\sqrt{\tfrac{2mE}{\hbar^2}}\tfrac{d}{2}\right)\\
\end{align}

I sum these two equations and I get:

\begin{align}
0&=2B\cos\left(\sqrt{\tfrac{2mE}{\hbar^2}}\tfrac{d}{2}\right)
\end{align}

And the possible solutions of this eq. are ##B=0## (this will help me find ##\psi##) but if ##B\neq 0## than even ##d\neq 0## or else we would get ##0=2B##. So if ##d\neq 0## than it must be true that:

\begin{align}
\sqrt{ \tfrac{2mE}{\hbar^2}} \tfrac{d}{2} &= (2N-1) \tfrac{\pi}{2} \longleftarrow N=1,2,3\dots\\
\sqrt{ \tfrac{2mE}{\hbar^2}} d &= (2N-1) \pi\\
&\!\!\!\!\!\!\boxed{E=\tfrac{(2N-1)^2\pi^2\hbar^2}{2md^2}}\\
E_1&=\pi^2\hbar^2/2md^2\\
E_2&=3^2E_1\\
E_3&=5^2E_1\\
&\dots
\end{align}

Now if I check two boxed equations I see that they are different! Even ##N## is defined the same for those two. Why did I get different energy equation by simply choosing a different interval (only the ground state ##E_1## is the same in both cases while ##E_2, E_3, E_4\dots## differ)? This means that my selection of an interval changes the particle? Please explain. I couldn't spot any mistake in my derivation. Can you?
 
Physics news on Phys.org
  • #2
Doing it this way (assuming B ≠ 0) you've gotten half the solutions. To get the other half, subtract one equation from the other and assume A ≠ 0.
 
  • #3
Thank you. If i substract these two equations i get:

\begin{align}
0&=-2A\sin\left(\sqrt{\tfrac{2mE}{\hbar^2}}\tfrac{d}{2}\right)
\end{align}

I can solve this with ##A=0## or ##d=0## but the later means that width of the well is 0 and can't be true which means ##A=0## is an only possible solution. Is this assumption ok? If i continue in this maner i get better values for the wavefunctions which must be

\begin{align}
\psi = B\cos\left(\sqrt{\tfrac{2mE}{\hbar^2}} x\right)
\end{align}

If i try to sum up all this in my head now. By summing the equations i got a ##B=0## solution, which now must not be used because if it is, both ##A=0## and ##B=0## which means ##\psi=0## and the particle wasn't supposed to be in the well. But it is! So i am left with only 3 possible equations which are ##A=0## the ##\psi = B\cos\left(\sqrt{\frac{2mE}{\hbar^2}} x\right)## and the one for energies - i mean the second boxed equation.

What now? Am i ready to normalize the wavefunction and Find ##B##?

PS: I can't forget the energy equation which is still not the same as for interval ##0<x<d##.
 
  • #4
71GA said:
Thank you. If i substract these two equations i get:

\begin{align}
0&=-2A\sin\left(\sqrt{\tfrac{2mE}{\hbar^2}}\tfrac{d}{2}\right)
\end{align}

I can solve this with ##A=0## or ##d=0## but the later means that width of the well is 0 and can't be true which means ##A=0## is an only possible solution. Is this assumption ok?
The solutions are √(2mE/ħ2) d/2 = Nπ.
 
  • Like
Likes 1 person
  • #5
Bill_K said:
The solutions are √(2mE/ħ2) d/2 = Nπ.

Oh right :) my solution was a bit trivial.

So if i use this i get another equation for energies which looks like:

$$\boxed{W=\frac{2N^2 \pi^2 \hbar^2}{md^2}}$$

I have now 2 equations for the energy levels on an interval ##-d/2<x<d/2## will the sum of those two return the same energy equation as i got for interval ##0<x<d##?
 
  • #6
[By the way, it would make things easier if each equation were in a separate block of LaTeX, enclosed in its own pair of tags. Then people could easily quote one equation at a time to make a point. I can't figure out how to do that, with your huge blocks of LaTeX.]

Here's another approach. Consider the general solution to the DE, with the coefficients A and B. Apply one of the boundary conditions, say psi(+d/2) = 0. The resulting equation (which you have a little further down the page) can be true if either

Case 1: A = 0 and cos(...) = 0. This gives you solutions of the form psi = B cos(kx), with the values of k (and E) determined by the condition cos(...) = 0.

or

Case 2: B = 0 and sin(...) = 0. This gives you solutions of the form psi = A cos (kx), with the values of k (and E) determined by the condition sin(...) = 0.

If you now start over again with the the general solution, and apply the other boundary condition (psi(-d/2) = 0), you get the same final result. The two boundary conditions are redundant because of the symmetry of the potential function.

Taken together, the allowed values of E for cases 1 and 2 are the same ones as for the other method (interval from 0 to d). The psi functions of course have somewhat different forms because of the shifted origin.
 
  • #7
There are also the following cases which don't enter into the final solution:

Case 3: A = 0 and B = 0, which is not useful, for obvious reasons!

Case 4: cos(...) = 0 and sin(...) = 0, which is impossible.
 
  • #8
I don't think you understood what i wanted to ask. I derived these two equations for an interval ##-d/2>x>d/2##:

$$W=\frac{2N^2 \pi^2 \hbar^2}{md^2}$$
$$W=\frac{(2N-1)^2 \pi^2 \hbar^2}{2md^2}$$

and this one using interval ##0>x>d##:

$$W=\frac{N^2 \pi^2 \hbar^2}{2md^2}$$

Energy levels should be equal but i still can't see the equality. What should i do with the first two to see that they are in fact the same as the later one?
 
  • #9
Write the first line as W = (2N)2π2ħ2/2md2. Then in the third line, N is either odd or even. If it's odd, that's the second line. If it's even, that's the first line.
 
  • Like
Likes 1 person
  • #10
Re-write the first equation as Bill indicated. Then write out a list of the values of 2N2, and of (2N-1)2, for N = 1, 2, 3...

See how they "fit together" with each other, and with the third equation?
 
  • #11
Thanks to both of you. I understand now.
 
  • #12
One more question. If i now want to derive wavefunctions for an interval ##-d/2 >x>d/2## i am left with two options. Either i take ##\psi = A\sin \left(\sqrt{\tfrac{2mW}{\hbar^2}} x\right)##, insert ##W=\frac{(2N-1)^2\pi^2\hbar^2}{2md^2}## and then normalise it OR i take ##\psi = B\cos \left(\sqrt{\tfrac{2mW}{\hbar^2}} x\right)##, and insert ##W=\frac{(2N)^2\pi^2\hbar^2}{2md^2}## and then normalise it.

I believe that in both cases i should get the same wavefunction? Please correct me if i am wrong.
 

1. What is the significance of selecting an interval in relation to energy eigenstates?

When discussing energy eigenstates, the interval refers to the range of values over which the particle's energy can exist. Therefore, the selection of this interval greatly affects the possible energy eigenstates that the particle can have.

2. How does the selection of an interval impact the energy eigenstates?

The selection of an interval can greatly limit or expand the possible energy eigenstates of a particle. For example, if a narrow interval is chosen, the particle will have a limited range of possible energy eigenstates. On the other hand, if a wide interval is selected, the particle can have a larger range of energy eigenstates.

3. Why is it impossible to predict energy eigenstates if the interval is not specified?

Without specifying the interval, the particle's energy can exist over an infinite range, making it impossible to predict or determine the energy eigenstates. The selection of an interval is essential in order to determine the energy eigenstates of a particle.

4. Can the selection of an interval affect the accuracy of predicting energy eigenstates?

Yes, the selection of an interval can greatly impact the accuracy of predicting energy eigenstates. If the selected interval does not encompass the actual energy range of the particle, the predicted energy eigenstates will be inaccurate.

5. Are there any limitations to the selection of an interval in determining energy eigenstates?

Yes, there are limitations to the selection of an interval. The interval must be physically possible and correspond to the actual energy range of the particle. Additionally, the selection of an interval may not always accurately predict the energy eigenstates, as there may be other factors at play such as external forces or interactions.

Similar threads

  • Quantum Physics
Replies
19
Views
1K
Replies
3
Views
378
Replies
5
Views
1K
Replies
2
Views
567
Replies
3
Views
857
Replies
2
Views
639
  • Quantum Physics
Replies
5
Views
511
  • MATLAB, Maple, Mathematica, LaTeX
Replies
1
Views
1K
Replies
1
Views
587
Replies
2
Views
4K
Back
Top